$$ \DeclareMathOperator{\vect}{vect} \DeclareMathOperator{\Ima}{Im} \DeclareMathOperator{\card}{card} \DeclareMathOperator{\Mat}{Mat} \DeclareMathOperator{\GL}{GL} \DeclareMathOperator{\rg}{rg} \DeclareMathOperator{\tr}{tr} \DeclareMathOperator{\Id}{Id} \DeclareMathOperator{\ch}{ch} \DeclareMathOperator{\sh}{sh} \DeclareMathOperator{\xhookrightarrow}{\raisebox{\depth}{\rotatebox{180}{\reflectbox{$\hookrightarrow$}}}} \renewcommand{\th}{\text{th}} \DeclareMathOperator{\argth}{argth} \DeclareMathOperator{\Arg}{Arg} \newcommand\Isom{\mathcal{I}\text{som}} \newcommand{\DL}[1]{développement limité à l'ordre $#1$ en zéro}$$
LMPrépa

Chapitre 11 : Continuité

Questions de cours

1. Donner la définition d'une fonction continue en $a \in\mathbb{R}$, avec une version quantifiée. Illustrer par 2 dessins (continue/discontinue). Donner des exemples de fonctions discontinues.

Définition :
Soit $f : I\to\mathbb{R}$ et $a\in I$
On dit que $f$ est continue en $a$ lorsque $f$ admet une limite finie en $a$. Cette limite est alors forcément $f(a)$.
Ainsi $f$ est continue en $a$ ssi $\displaystyle{} f(x)\underset{x\rightarrow a}\longrightarrow f(a)$ $$\forall \varepsilon > 0, \exists \delta > 0, \forall x\in I, (|x-a|\leq \delta)\Rightarrow (|f(x)-f(a)|\leq \varepsilon)$$

2. Fonctions lipschitzienne : définition, 2 exemples et un contre-exemple continu.
SAVOIR REFAIRE : Montrer que toute fonction lipschitzienne est continue.

Définition :
Soit $f : I\to \mathbb{R}$. On dit que $f$ est lipschitzienne sur $I$ lorsqu'il existe un certain $\lambda > 0$ tq $$\forall (x,y)^2\in I^2, |f(x)-f(y)|\leq \lambda|x-y|$$
Preuve :
On suppose que $f$ est $\lambda-$lipschitzienne sur $I$ où $\lambda > 0$
Soit $a\in I$. Montrons que $f$ est continue en $a$
Soit $\varepsilon > 0$, on pose $\displaystyle{}\delta = \frac{\varepsilon}{\lambda}$
Soit $x\in I$ tq $|x-a|\leq \delta$
Alors $|f(x) - f(a)|\leq \lambda|x-a|\leq \lambda\delta = \varepsilon$
i.e. $f$ est continue en $a$. Exemples :

$\begin{array}{lll} 1. \text{Les fonctions affines }f : & \mathbb{R} \to \mathbb{R} & \text{sont lipschitziennes sur }\mathbb{R}\\ & x\mapsto ax+b & \end{array}$
En effet si $(x,y)\in\mathbb{R}^2$, alors \begin{eqnarray*} |f(x) - f(y)| &=& |ax+b - (ay+b)|\\ &=& |a||x-y| \end{eqnarray*} Donc $f$ est $|a|-$lipschitzienne sur $\mathbb{R}$ (Donc continue sur $\mathbb{R}$)

$\begin{array}{lll} \text{2. Les valeurs absolues }f : & \mathbb{R} \to \mathbb{R} & \text{sont lipschitziennes sur }\mathbb{R}\\ & x\mapsto |x| & \end{array}$
En effet si $(x,y)\in\mathbb{R}^2$ alors $|f(x)-f(y)| = ||x|-|y||\leq |x-y|$
Donc $f$ est $1-$lipschitzienne sur $\mathbb{R}$ (Donc continue sur $\mathbb{R}$)

$\begin{array}{lll} 3. f : & \mathbb{R}\to\mathbb{R} & \text{ est continue sur }\mathbb{R} \text{ mais non lipschitzienne sur }\mathbb{R}\\ & x\mapsto x^2 \end{array}$
$\bullet$ $f$ est continue (cf théorèmes opératoires)
$\bullet$ $f$ n'est pas lipschitzienne sur $\mathbb{R}$

En effet par l'absurde, soit $\lambda > 0$ tq $\begin{array}{lccc} \forall(x,y)\in \mathbb{R}^2, & |f(x)-f(y)| & \leq & \lambda|x-y|\\ \forall(x,y)\in \mathbb{R}^2, & |x^2-y^2| & \leq & \lambda|x-y| \end{array}$
CP : $y = 0$, $\forall x\in\mathbb{R}$, $|x|^2 \leq \lambda|x|$
Si $x > 0$, $|x| < \lambda$ faux avec $x = \lambda + 1$
Contradiction.

3. Donner la caractérisation séquentielle de la continuité. A quoi cela peut-il servir ?

Théorème :
Soient $f : I\to\mathbb{R}$, $a\in I$ alors
$f$ est continue en $a$ $\Leftrightarrow$ pour toute suite $(x_n)$ de $I$ qui converge vers $a$, $f((x_n))$ converge vers $f(a)$

4. Donner un exemple de fonction prolongeable par continuité et un contre-exemple

Définition :
Soit $f : I\backslash \{ a \}\to\mathbb{R}$ où $a\in\bar I$
On suppose que $f$ est continue sur $I\backslash \{ a \}$
$\begin{array}{l|l|l} \text{On suppose que }f\text{ admet un prolongement }&\text{ continu }&\text{en }a\text{ lorsque }f(x)\xrightarrow[x\rightarrow a]{} \ell\in\mathbb{R}\\ & \text{par continuité}& \end{array}$
Ce prolongement continue est alors unique et obtenu en posant $f(a) = \ell$

5. SAVOIR REFAIRE : soit $f : \mathbb{R} \to\mathbb{R}$ additive et continue. Montrer que $f$ est une homothétie


1.a. Soient $x$ et $y$ dans $\mathbb{R}$
$f(x+y) - f(y) = f(x + y - y)$ car $f$ est additive
i.e. $f(x-y) + f(y) = f(x)$
Donc $f(x-y) = f(x)-f(y)$

1.b. Par récurrence sur $n\in\mathbb{N}$, on fixe $x\in\mathbb{R}$
Initialisation : $n = 0$
$x = y = 0$ donne $f(0+0) = f(0)+f(0)$ i.e. $f(0) = 0$
Donc $f(0\times x) = f(0) = 0 = 0\times f(x)$
Hérédité :
Soit $n\in\mathbb{N}$ tq $f(nx) = nf(x)$ \begin{eqnarray*} f((n+1)x) &=& f(nx+x)\\ &=& f(nx)+f(x)\hspace{20pt}\text{car }f\text{ est additive}\\ &=& n(f(x)) + f(x) \hspace{20pt}\text{par hypothèse de récurrence}\\ &=& (n+1)f(x) \end{eqnarray*} Ce qui achève la récurrence.

1.c. Soit $n\in\mathbb{Z}$.
Si $n \geq 0$, OK (cf 1.b.)
Si $n\leq 0$, alors on pose $p = -n\in\mathbb{N}$
$f(nx) = f(-px) = f(0-px)$
D'après le 1.a., $f(nx) = f(0) - f(px) = -pf(x) = nf(x)$

1.d. Soit $r\in\mathbb{Q}$.
D'ou $(p,q)\in\mathbb{Z} \times \mathbb{N}^*$ tq $\displaystyle{}r = \frac{p}{q}$
Soit $x\in\mathbb{R}$
Mq $\displaystyle{}f\left( rx \right) = rf(x)$
Mq $\displaystyle{}f\left( \frac{p}{q}x \right) = \frac{p}{q}f(x)$
Or $\displaystyle{}qf\left( \frac{p}{q}x \right) = f(px) = pf(x)$
Ainsi $\displaystyle{}f\left( \frac{p}{q}x \right) = \frac{p}{q}f(x)$

Remarque : avec $x=1$ on obtient $\forall r\in\mathbb{Q}, f(r) = rf(1)$, donc si on pose $\lambda = f(x)$, $\forall r\in\mathbb{Q}$, $f(r) = \lambda r$ i.e. $f$ est linéaire sur $\mathbb{Q}$

2. On suppose maintenant que $f$ est continue sur $\mathbb{R}$.
On pose toujours $\lambda = f(1)$
Soit $x\in\mathbb{R}$
$\mathbb{Q}$ est dense dans $\mathbb{R}$, d'où une suite $(r_n)$ de rationnels $r_n \xrightarrow[n\rightarrow +\infty]{}x$
Si $n\in\mathbb{N}$, $f(r_n) = \lambda r_n$ quand $n$ tend vers $+\infty$ $f(x) = \lambda x$ car $f$ est continue en $x$ (continuité séquentielle)
Conclusion : $\forall x \in\mathbb{R}$, $f(x) = \lambda x$

6. Citer le théorème des valeurs intermédiaires.

Théorème : TVI version ensembliste
L'image d'un intervalle par une fonction continue est encore un intervalle
Théorème : TVI version pratique
Si $f\to\mathbb{R}$ est continue sur l'intervalle $I$
Si $a < b$ dans $I$
Alors toute valeur intermédiaire entre $f(a)$ et $f(b)$ est prise par $f$ i.e. elle est dans l'image de $f$

7. SAVOIR REFAIRE : montrer que $x\mapsto 2x^5-7x^3 + 2x^2 - 1$ s'annule sur $\mathbb{R}$


$P(x)\underset{+\infty}\sim 2x^5$, donc $P(x)\xrightarrow[x\rightarrow+\infty]{}+\infty$
D'où un certain $b > 0$ tq $P(b) > 0$
$P(x)\xrightarrow[x\rightarrow-\infty]{}-\infty$
D'où un certain $a < 0$ tq $P(a) < 0$
D'où $c\in\mathbb{R}$ tq $P(c) = 0$ d'après le théorème des valeurs intermédiaires.

8. SAVOIR REFAIRE : ("Petit BROUWER") : Soit $f : [a;b] \to [a;b]$ continue. Montrer que $f$ a un point fixe.


Preuve :
$\begin{array}{ll} \text{On envisage la fonction auxiliaire }g : & [a;b]\to\mathbb{R}\\ & x \mapsto f(x) - x \end{array}$
Il suffit de montrer que $g$ s'annule
$g(a) = f(a)-a \geq 0$ car $f(a)\in Imf\subset [a;b]$ donc $f(a)\geq a$
$g(b) = f(b)-b \leq 0$ car $f(b)\in Imf\subset [a;b]$ donc $f(b)\leq b$

Or $g$ est continue
Donc d'après le TVI, $g$ s'annule sur $[a;b]$
D'où $c\in[a;b]$ tq $g(c) = 0$ i.e. $f(c) = c$

9. Que dire d'une fonction continue sur un segment ? Puis : que signifie "elle atteint ses bornes" ?

Théorème :
Toute fonction continue sur un segment est bornée et atteint ses bornes.

$\begin{array}{l|l} \text{Soient }& a < b \text{ dans }\mathbb{R}\\ & f:[a;b]\to\mathbb{R}\text{ continue} \end{array}$
Alors $f$ est borné et $\underset{[a;b]}\sup(f)=M$ et $\underset{[a,b]}\inf(f)=m$ sont atteints i.e. $\exists x_0\in[a;b], f(x_0) = \underset{[a;b]}\sup(f)=M$ et $\exists x_1\in[a;b], f(x_1) = \underset{[a;b]}\inf(f)=m$
i.e. $\underset{[a;b]}\sup(f) = \underset{[a;b]}\max(f)$ et $\underset{[a;b]}\inf(f) = \underset{[a;b]}\min(f)$

10. SAVOIR REFAIRE : soit $f : \mathbb{R}_+\to\mathbb{R}$ continue et telle que $f\xrightarrow[+\infty]{}\ell\in\mathbb{R}$. Montrer que $f$ est bornée. Atteint-elle ses bornes ?


Preuve :
$f\xrightarrow[+\infty]{}\ell\in\mathbb{R}$
Donc $\forall \varepsilon > 0, \exists A > 0, \forall x \in\mathbb{R}_+, (x\geq A)\Rightarrow(|f(x)-\ell| \leq \varepsilon)$
On choisit $\varepsilon = 1$, d'où $A > 0$ tq $\forall < \geq A, |f(x) - \ell|\leq 1$
si $x\geq A, |f(x)| = |f(x)-\ell+\ell|\leq |f(x) - \ell|+|\ell|$
$\forall w \geq A, |f(x)|\leq [\ell|+1$
Par ailleurs $f$ est continue sur le segment $[0;A]$
Donc d'après le cours, $\exists M_1 > 0, \forall x \in [0;A], f(x) \leq M_1$
On pose $M = \max(M_1, |l|+1)$
Ainsi $\forall c\in\mathbb{R}_+, f(x)\geq M$
Donc $f$ est bornée.

11. Que dire d'une fonction continue et injective sur un intervalle ?


Toute fonction continue et injective sur un intervalle est strictement monotone.

12. Énoncer le théorème de la bijection continue.

Théorème : bijection continue
Soit $f : I\to\mathbb{R}$ une fonction strictement monotone et continue.
$\begin{array}{ll} \text{Alors } & 1. f(I)\text{ est un intervalle (noté }J)\\ & 2. f\text{ réalise une bijection de }I\text{ sur }J\\ & 3.\text{ la bijection réciproque, notée }f^{-1}\text{ est automatiquement continue de }J\text{ sur }I \end{array}$

13. SAVOIR REFAIRE : soit $g : J\to\mathbb{R}$ strictement monotone telle que $g(J)$ soit un intervalle. Montrer que $g$ est continue sur $J$

Lemme :
Soit $g : J\to\mathbb{R}$ (où $J$ est intervalle non trivial de $\mathbb{R}$) tq $g$ est strictement monotone et $f(J)$ est un intervalle.
Alors $g$ est continue en $J$.
Preuve :
Soit $a\in J$, montrons que $g$ est continue en $a$.
Supposons que $\overset{\circ}J$ (point intérieur)
D'après le TLM, $g$ admet une limite à gauche et une limite à droite en $a$.
Supposons que $g$ est strictement croissante.
D'après le TLM, $g(a-0) \leq g(a) \leq g(a+0)$
$g$ est continu en $a$ ssi $g(a-0) = g(a) = g(a+0)$ $g$ est strictement croissante
$\begin{array}{ll} \text{donc } & g(J \cap ]-\infty;a[)\subset ]-\infty ; g(a-0)[\\ \text{i.e. } & \forall x \in J_1, (x < a) \Rightarrow (g(x)\leq g(a-0))\\ & g(J\cap ]a ; +\infty[)\subset ]g(a+0) ; +\infty[\\ \text{i.e. } & \forall x\in J_2, (x > 0) \Rightarrow (g(x) \geq g(a+0)) \end{array}$
On a $J = J_1 \bigsqcup \{a\}\bigsqcup J_2$
Donc $g(J) = g(J_1) \cup \{a\} \cup g(J_2)$ (par un intervalle si $g(a-0) < g(a)$ ou si $g(a+0) > g(a)$)
Donc $g(a-0) = g(a) = g(a+0)$
Si $a\notin \underset{\circ}J$ EXO

14. Donner la définition d'une fonction uniformément continue et quelques exemples

Définition :
On dit qu'une fonction $f : I\to \mathbb{R}$ est uniformément continue sur $I$ lorsque : $$\forall \varepsilon > 0, \exists\delta > 0, \forall(x,y)\in I^2, (|x-y|\leq \delta) \Rightarrow (|f(x) - f(y)| \leq \varepsilon)$$

15. Donner 3 exemples de fonction non uniformément continue, mais continues.


$\begin{array}{lllll} \text{1. }f : & \mathbb{R}_+^* & \to & \mathbb{R} & \text{est continue mais pas uniformément continue sur }\mathbb{R}_+^*\\ & x & \mapsto & \frac{1}{x} \end{array}$
Idée : $f$ explose brutalement en 0
On psoe $\displaystyle{}x_n = \frac{1}{n+1}$ et $\displaystyle{}y_n = \frac{1}{n}$
Ainsi $(x_n-y_n)\xrightarrow[n\rightarrow+\infty]{}0$
Cependant $|f(x_n) - f(y_n)| = |n+1-n| = 1$ que ne tend pas vers 0
Donc $f$ n'est pas uniformément continue sur $\mathbb{R}_+^*$

$\begin{array}{lcccll} 2. f : & \mathbb{R} & \to & \mathbb{R} & \text{ est }C^{\circ}&\text{ mais pas uniformément continue sur }\mathbb{R}\\ & x & \mapsto & x^2 \end{array}$
Moralement elle tend trop vite vers $+\infty$ et $-\infty$

On pose $x_n = \sqrt{n+1}$ et $y_n = \sqrt{n}$
$\displaystyle{}|x_n-y_n| = \sqrt{n+1}-\sqrt{n} = \frac{1}{\sqrt{n+1}-\sqrt{n}}\xrightarrow[n\rightarrow+\infty]{}0$
Cependant $|f(x_n) - f(y_n)| = \left( \sqrt{n+1}\right)^2 - \left( \sqrt{n} \right)^2 = n + 1 - n = 1$ que ne tend pas vers 0
Donc $f$ n'est pas uniformément continue sur $\mathbb{R}$

3. Vrai-Faux : "$f C^{\circ}$ est bornée $\Rightarrow f$ uniformément continue"
Faux : Contre exemple : "Serpent frénétique"
$\begin{array}{lcccl} f : & ]0;1] & \to & \mathbb{R} & \text{est bornée mais pas uniformément continue sur }]0;1]\\ & x & \mapsto & \sin\left(\frac{1}{x}\right) \end{array}$
On choisit $\displaystyle{}x_n = \frac{1}{2\pi n}$ et $\displaystyle{}y_n = \frac{1}{\frac{\pi}{n}+2\pi n}$
$(x_n - y_n)\xrightarrow[n\rightarrow+\infty]{}0$
Cependant $\displaystyle{}|f(x_n) - f(y_n)| = \sin(2\pi n)-\sin\left( \frac{\pi}{2}+2\pi n \right) = 1$ qui ne tend pas vers 0

16. Donner les rapports entre fonction : continue/lipschitzienne/uniformément continue.

Théorème : Comparaison de trois notions
Soit $f : I\to\mathbb{R}$ alors
$f$ lipschitzienne sur $I^\Rightarrow f$ est uniformément continue sur $I \Rightarrow f$ est $C^{\circ}$ sur $I$
sans réciproque en général.

17. Énoncer le théorème de Heine.

Théorème :
Toute fonction continue sur un segment est uniformément continue.
Page précédente :
QC 10 : Limite
Page suivante :
QC 12 : Dérivabilité